1975 Canadian MO Problems/Problem 1

Revision as of 15:47, 5 August 2016 by Memc38123 (talk | contribs) (Problem 1)

Problem 1

Simplify \[\left(\frac{1\cdot2\cdot4+2\cdot4\cdot8+\cdots+n\cdot2n\cdot4n}{1\cdot3\cdot9+2\cdot6\cdot18+\cdots+n\cdot3n\cdot9n}\right)^{1/3}\].

Solution

None yet!

1975 Canadian MO (Problems)
Preceded by
First question
1 2 3 4 5 6 7 8 Followed by
Problem 2